Complex decimal problem - any shortcuts?

This topic has expert replies
User avatar
Junior | Next Rank: 30 Posts
Posts: 11
Joined: Thu Feb 16, 2012 4:40 am
Thanked: 1 times
Q. 199 on OG 13

(0.99999999 ÷ 1.0001) - (0.99999991 ÷ 1.0003) =

(A) 10^-8
(B) 3(10^-8)
(C) 3(10^-4)
(D) 2(10^-4)
(E) 10^-4

The right answer is D. Is there any shortcut for this question? The solution in the OG is tedious.

User avatar
Legendary Member
Posts: 588
Joined: Sun Oct 16, 2011 9:42 am
Location: New Delhi, India
Thanked: 130 times
Followed by:9 members
GMAT Score:720

by rijul007 » Tue Sep 11, 2012 8:11 am
rachitakapoor wrote:Q. 199 on OG 13

(0.99999999 ÷ 1.0001) - (0.99999991 ÷ 1.0003) =

(A) 10^-8
(B) 3(10^-8)
(C) 3(10^-4)
(D) 2(10^-4)
(E) 10^-4

The right answer is D. Is there any shortcut for this question? The solution in the OG is tedious.
(0.99999999 ÷ 1.0001) - (0.99999991 ÷ 1.0003)
(9999.9999/10001) - (9999.9991/10003)
(9999 * 1.0001/10001) - (9997 * 1.0003/10003)
(9999 * 10^-4) - (9997 * 10^-4)
0.9999 - 0.9997
0.0002

Option D

User avatar
GMAT Instructor
Posts: 15539
Joined: Tue May 25, 2010 12:04 pm
Location: New York, NY
Thanked: 13060 times
Followed by:1906 members
GMAT Score:790

by GMATGuruNY » Tue Sep 11, 2012 8:41 am
rachitakapoor wrote:Q. 199 on OG 13

(0.99999999 ÷ 1.0001) - (0.99999991 ÷ 1.0003) =

(A) 10^-8
(B) 3(10^-8)
(C) 3(10^-4)
(D) 2(10^-4)
(E) 10^-4

The right answer is D. Is there any shortcut for this question? The solution in the OG is tedious.
Approach 1:

(9999.9999)/(10001) - (9999.99991)(10003)

= (10^-4)(99999999/10001) - (10^-4)(99999991/10003)

= (10^-4) (99999999/10001 - 99999991/10003)

Only C, D and E include 10^-4.
Thus, the correct answer must be C, D or E, implying three possibilities:
99999999/10001 - 99999991/10003 = 3.
99999999/10001 - 99999991/10003 = 2.
99999999/10001 - 99999991/10003 = 1.

Since their difference is an integer, the quotients here are almost certainly integer values themselves.
Focus on the UNITS DIGITS:
The units digit of 99999999/10001 must be 9, since 10001 must be multiplied by a units digit of 9 to yield 99999999.
The units digit of 99999991/10003 must be 7, since 10003 must be multiplied by a units digit of 7 to yield 99999991.
Since 9-7=2:
99999999/10001 - 99999991/10003 = 2.

The correct answer is D.

Approach 2:

(9999.9999)/10001 - (9999.9991)/10003

= [10003(9999.9999) - 10001(9999.9991)] / (10001)(10003)

≈ [10003(10^4) - 10001(10^4)] / (10^4)(10^4)

≈ (10003-10001)/(10^4)

≈ 2(10^-4).

The correct answer is D.

Approach 3:
Try a simpler version of the problem.

.99/1.1 - .91/1.3

= 99/110 - 91/130

= ( 99/11 - 91/13) (1/10)

= (9-7)(10^-1)

= 2(10^-1).

This result suggests that the correct answer almost certainly is D.
Private tutor exclusively for the GMAT and GRE, with over 20 years of experience.
Followed here and elsewhere by over 1900 test-takers.
I have worked with students based in the US, Australia, Taiwan, China, Tajikistan, Kuwait, Saudi Arabia -- a long list of countries.
My students have been admitted to HBS, CBS, Tuck, Yale, Stern, Fuqua -- a long list of top programs.

As a tutor, I don't simply teach you how I would approach problems.
I unlock the best way for YOU to solve problems.

For more information, please email me (Mitch Hunt) at [email protected].
Student Review #1
Student Review #2
Student Review #3

User avatar
Junior | Next Rank: 30 Posts
Posts: 11
Joined: Thu Feb 16, 2012 4:40 am
Thanked: 1 times

by rachitakapoor » Tue Sep 11, 2012 11:46 pm
rijul007 wrote:
rachitakapoor wrote:Q. 199 on OG 13

(0.99999999 ÷ 1.0001) - (0.99999991 ÷ 1.0003) =

(A) 10^-8
(B) 3(10^-8)
(C) 3(10^-4)
(D) 2(10^-4)
(E) 10^-4

The right answer is D. Is there any shortcut for this question? The solution in the OG is tedious.
(0.99999999 ÷ 1.0001) - (0.99999991 ÷ 1.0003)
(9999.9999/10001) - (9999.9991/10003)
(9999 * 1.0001/10001) - (9997 * 1.0003/10003)
(9999 * 10^-4) - (9997 * 10^-4)
0.9999 - 0.9997
0.0002

Option D
Many thanks, rijul007! :)

User avatar
Junior | Next Rank: 30 Posts
Posts: 11
Joined: Thu Feb 16, 2012 4:40 am
Thanked: 1 times

by rachitakapoor » Tue Sep 11, 2012 11:51 pm
GMATGuruNY wrote:
rachitakapoor wrote:Q. 199 on OG 13

(0.99999999 ÷ 1.0001) - (0.99999991 ÷ 1.0003) =

(A) 10^-8
(B) 3(10^-8)
(C) 3(10^-4)
(D) 2(10^-4)
(E) 10^-4

The right answer is D. Is there any shortcut for this question? The solution in the OG is tedious.
Approach 1:

(9999.9999)/(10001) - (9999.99991)(10003)

= (10^-4)(99999999/10001) - (10^-4)(99999991/10003)

= (10^-4) (99999999/10001 - 99999991/10003)

Only C, D and E include 10^-4.
Thus, the correct answer must be C, D or E, implying three possibilities:
99999999/10001 - 99999991/10003 = 3.
99999999/10001 - 99999991/10003 = 2.
99999999/10001 - 99999991/10003 = 1.

Since their difference is an integer, the quotients here are almost certainly integer values themselves.
Focus on the UNITS DIGITS:
The units digit of 99999999/10001 must be 9, since 10001 must be multiplied by a units digit of 9 to yield 99999999.
The units digit of 99999991/10003 must be 7, since 10003 must be multiplied by a units digit of 7 to yield 99999991.
Since 9-7=2:
99999999/10001 - 99999991/10003 = 2.

The correct answer is D.

Approach 2:

(9999.9999)/10001 - (9999.9991)/10003

= [10003(9999.9999) - 10001(9999.9991)] / (10001)(10003)

≈ [10003(10^4) - 10001(10^4)] / (10^4)(10^4)

≈ (10003-10001)/(10^4)

≈ 2(10^-4).

The correct answer is D.

Approach 3:
Try a simpler version of the problem.

.99/1.1 - .91/1.3

= 99/110 - 91/130

= ( 99/11 - 91/13) (1/10)

= (9-7)(10^-1)

= 2(10^-1).

This result suggests that the correct answer almost certainly is D.
Thanks a tonne for all the approaches! Looking at various approaches does help a lot and I'm gonna try finding atleast 2 of them for the tough problems.

GMAT/MBA Expert

User avatar
GMAT Instructor
Posts: 16207
Joined: Mon Dec 08, 2008 6:26 pm
Location: Vancouver, BC
Thanked: 5254 times
Followed by:1268 members
GMAT Score:770

by Brent@GMATPrepNow » Wed Sep 12, 2012 6:47 am
rachitakapoor wrote: Looking at various approaches does help a lot and I'm gonna try finding at least 2 of them for the tough problems.
Yes!! This is an awesome strategy that all students should practice while preparing for the GMAT (for all math questions).

Cheers,
Brent
Brent Hanneson - Creator of GMATPrepNow.com
Image

User avatar
Junior | Next Rank: 30 Posts
Posts: 11
Joined: Sat Oct 06, 2012 1:55 pm
Followed by:1 members
GMAT Score:690

by KateG1302 » Sat Oct 20, 2012 5:11 am
rijul007 wrote:
rachitakapoor wrote:Q. 199 on OG 13

(0.99999999 ÷ 1.0001) - (0.99999991 ÷ 1.0003) =

(A) 10^-8
(B) 3(10^-8)
(C) 3(10^-4)
(D) 2(10^-4)
(E) 10^-4

The right answer is D. Is there any shortcut for this question? The solution in the OG is tedious.
(0.99999999 ÷ 1.0001) - (0.99999991 ÷ 1.0003)
(9999.9999/10001) - (9999.9991/10003)
(9999 * 1.0001/10001) - (9997 * 1.0003/10003)
(9999 * 10^-4) - (9997 * 10^-4)
0.9999 - 0.9997
0.0002

Option D
Hi,

sorry but I just don't understand how to get from:

(9999.9999/10001) - (9999.9991/10003)

to

(9999 * 1.0001/10001) - (9997 * 1.0003/10003)


I see how the third approach works (i.e. using "simplified" numbers) but I am worried that I will not recognise this pattern during the real test, so I'd like to understand the calculations properly.

Thanks in advance !

Kate

User avatar
Junior | Next Rank: 30 Posts
Posts: 11
Joined: Sat Oct 06, 2012 1:55 pm
Followed by:1 members
GMAT Score:690

by KateG1302 » Tue Oct 23, 2012 4:59 am
Hi all,


can nobody help with the above?

Thanks a lot!
Kate

GMAT/MBA Expert

User avatar
GMAT Instructor
Posts: 16207
Joined: Mon Dec 08, 2008 6:26 pm
Location: Vancouver, BC
Thanked: 5254 times
Followed by:1268 members
GMAT Score:770

by Brent@GMATPrepNow » Tue Oct 23, 2012 6:04 am
KateG1302 wrote: Hi,

sorry but I just don't understand how to get from:

(9999.9999/10001) - (9999.9991/10003)

to

(9999 * 1.0001/10001) - (9997 * 1.0003/10003)


I see how the third approach works (i.e. using "simplified" numbers) but I am worried that I will not recognise this pattern during the real test, so I'd like to understand the calculations properly.

Thanks in advance !

Kate
Hi Kate,

The step from (9999.9991/10003)
to
(9997 * 1.0003/10003)
is a gigantic step. Don't worry, the vast majority of students won't see it either.

I believe that the test maker wants us to recognize that both 9999.9999 and 9999.9991 can be rewritten as differences of squares.

First, 0.99999999 = 1 - 0.00000001
= (1 - 0.0001)(1 + 0.0001)

Similarly, 9999.9991 = 1 - 0.00000009
= (1 - 0.0003)(1 + 0.0003)

Original question: 0.99999999/1.0001 - 0.99999991/1.0003
= (1 - 0.0001)(1 + 0.0001)/(1.0001) - (1 - 0.0003)(1 + 0.0003)/(1.0003)
= (1 - 0.0001)(1.0001)/(1.0001) - (1 - 0.0003)(1.0003)/(1.0003)
= (1 - 0.0001) - (1 - 0.0003)
= 0.0002
= 2 x 10^(-4) = D

Cheers,
Brent
Brent Hanneson - Creator of GMATPrepNow.com
Image